LSAT and Law School Admissions Forum

Get expert LSAT preparation and law school admissions advice from PowerScore Test Preparation.

User avatar
 Dave Killoran
PowerScore Staff
  • PowerScore Staff
  • Posts: 5852
  • Joined: Mar 25, 2011
|
#87956
Complete Question Explanation
(The complete setup for this game can be found here: lsat/viewtopic.php?f=159&t=11958)

The correct answer choice is (C)

This question asks you to identify the additional condition, that when added to the existing conditions, will result in only a single solution to the game. As is normally the case with this type of question in a Linear game, search for an answer that places a “front” variable near the “back,” or an answer that places a “back” variable near the “front.” Second, search for one of the above scenarios that features a variable that appears in more than one rule or that links to two or more variables.

The scenarios described above are usually restricted, and placing a variable in one of the situations above usually forces other variables into a single position.

Let’s look at each answer choice using the analysis pattern above:

Answer choice (A) features H, a variable that would be more of a “back” variable, and places H at 4, a position further towards the back. H also appears in only one rule. Thus, this is not an answer choice likely to yield the correct answer.

Answer choice (B) features J, which appears in two rules. But, J is more of a “front” variable, and this answer choice places J in the first slot.

Answer choice (C) also features J, and places J in a “back” position. At first glance, this answer choice has potential.

Answer choice (D) features L, a variable that appears in two rules, but is neither a “front” nor “back” variable. Although placing L in the first audition forces G to be second and J to be third, there does not appear to be any further consequences, and thus there appears to be several possible options for the remaining variables.

Answer choice (E) also features L, but by placing L second, there are even more options created than in answer choice (D).

Thus, at first glance, answer choice (C) has the greatest potential. And, indeed, when J auditions at 5, H must audition at 6, and L must audition at 3. K must then audition at 4, forcing G to audition at 2 and F to audition at 1:

G1-Q4-d1.png

Thus, answer choice (C) is correct.
You do not have the required permissions to view the files attached to this post.
 christos
  • Posts: 1
  • Joined: Aug 08, 2016
|
#78016
Solution to Q4?
 David Boyle
PowerScore Staff
  • PowerScore Staff
  • Posts: 836
  • Joined: Jun 07, 2013
|
#78017
Hello christos,

The template for question 4 is fglkjh.

Hope this helps,
David
 ray57
  • Posts: 9
  • Joined: Aug 09, 2019
|
#67646
Hello,

This question took a very long time for me to figure out because I think I missed a deduction. What is the deduction that had to be made to answer this one more quickly, or did I just miss something here? Thanks for your help.
Ray
 Rachael Wilkenfeld
PowerScore Staff
  • PowerScore Staff
  • Posts: 1358
  • Joined: Dec 15, 2011
|
#67842
Hi Ray,

I'm not sure what deductions you made and what you missed, but I can tell you that you could have had the correct set up, but still needed some time on this question. The key here, as it is with many local questions, is to strategically decide which answer choices you should try to draw out. In order to do that, you need to think about where the game is limited.

We have a lot of limitations 1) after J and after G, 2) with J and L spacing, and 3) with L and G spacing. That makes answer choice (A) an unlikely contender. I won't eliminate it, but I won't start with drawing it out either because it doesn't really impact most of those relationships. Answer choice (B) isn't likely to be correct either. I see it starts with J in 1. J is a limited variable, but starting with J in slot 1 is a really open sort of set up. We know L is in 3, but that's about it. For me, I wouldn't diagram this one out either, though you could if you so desired.

Answer choice (C) is starting to look limited. It puts J in 5, and we know someone goes after J. That means off the bat, I'll have three variables to place (J, L, and H). I'll draw that out as a likely contender. If J is in 5, H is after J in 6. L is two slots before J in 3. Now it gets interesting. Theoretically G could go before OR after L. But if we put G after L, there's no slot for K. So now we know G is in 2, K is in 4, and that leaves F in one. Our order for this answer choice is fully set as F, G, L, K, J, H.

Answer choice (D) is another one I wouldn't bother to draw out. I see that L is in 1, which I know means G is in 2, and L is in 3. But then it's a big old question for 4, 5, and 6.

Answer choice (E) puts L in 2. This is even LESS helpful than answer choice (D) was, because now, G can be either 1/3. I wouldn't draw it out either.

In summary, there's not really a missing inference likely, but an incorrect strategy. You only want to draw things out if you need to draw them. I didn't eliminate answer choice (A), (B), (D), and (E) without drawing them out, but I did want to pick the most likely answer to draw. I saw that as answer choice (C).

Hope that helps!
Rachael

Get the most out of your LSAT Prep Plus subscription.

Analyze and track your performance with our Testing and Analytics Package.